12
$\begingroup$

I posted this in the mathematical forums. Maybe you will help me. I found an hard article Link of yang huang and luttinger. The authors begins with the sum: $$\frac{{E_2 }}{{E_0 }} = \frac{{16\pi ^2 a^2 \lambda ^2 }}{{V^2 }}\sum\limits_{} {'\frac{{\langle n_\alpha \rangle \langle n_\gamma \rangle \langle n_\lambda \rangle }}{{\frac{1}{2}\left( {k_\alpha ^2 + k_\beta ^2 - k_\gamma ^2 - k_\lambda ^2 } \right)}}\delta \left( {\vec k{}_\alpha + \vec k{}_\beta - \vec k{}_\gamma - \vec k{}_\lambda } \right)} $$ which represents the second order perturbation term of the energy of a bose gas. ${\langle n_\alpha \rangle }$ is: $$\langle n_\alpha \rangle = \sum\limits_{n = 0}^\infty {n\left( {ze^{ - \beta \varepsilon _\alpha } } \right)} ^n /\sum\limits_{n = 0}^\infty {\left( {ze^{ - \beta \varepsilon _\alpha } } \right)} ^n = \frac{{ze^{ - \beta \varepsilon _\alpha } }}{{1 - ze^{ - \beta \varepsilon _\alpha } }} $$ In the sum the terms with a vanishing denominator are omitted. There is also the restriction ${\vec k{}_\alpha \ne \vec k{}_\beta }\ \ $ and ${\vec k{}_\gamma \ne \vec k{}_\lambda }\ \ $. Now the passage that is unclear to me is the passage to the integral: $$\frac{{E_2 }}{{E_0 }} = \frac{{16\pi a^2 \lambda ^2 }}{{V^2 }}\left( {\frac{{4V^3 }}{{\pi ^3 \lambda ^5 }}} \right)\sum\limits_{i,j,k}^\infty {\frac{{z^{j + k + l} }}{{\left( {j + k + l} \right)^{1/2} \left( {j - k} \right)l}}} \frac{{\partial J}}{{\partial u}} $$ where: $$J = \int\limits_0^\infty {\int\limits_0^\infty {dqdq\frac{{\cosh \left( {upq} \right)}}{{q^2 - p^2 }}} } e^{ - vq^2 - wp^2 } $$ and: $$u = \frac{{\hbar ^2 \beta }}{{2m}}\left( {\frac{{2\left( {j - k} \right)l}}{{j + k + l}}} \right) $$ $$v = \frac{{\hbar ^2 \beta }}{{2m}}\left( {\frac{{\left( {j + k} \right)l}}{{j + k + l}}} \right) $$ $$w = \frac{{\hbar ^2 \beta }}{{2m}}\left( {\frac{{\left( {j + k} \right)l + 4jk}}{{j + k + l}}} \right) $$ $\frac{{\partial J}}{{\partial u}}$ is calculated here: https://math.stackexchange.com/questions/63534/difficult-integral Maybe we don't have to do thermal average of n and we need : $$n = \frac{1}{{z^{ - 1} e^{\varepsilon /kT} - 1}} = \sum\limits_{n = 1}^\infty {\left( {ze^{ - \varepsilon /kT} } \right)^l } $$ so: $$\sum\limits_{} {'... = \sum\limits_{i,j,k = 1}^\infty {\sum\limits_{} ' } } z^{ijk} \frac{{e^{ - \frac{{\hbar ^2 k_a^2j }}{{2m}}} e^{ - \frac{{\hbar ^2 k_\gamma ^2k }}{{2m}}} e^{ - \frac{{\hbar ^2 k_\lambda ^2l }}{{2m}}} }}{{\frac{1}{2}\left( {k_a^2 + k_b^2 - k_\gamma ^2 - k_\lambda ^2 } \right)}} $$ This explain the factor $z^{ijk}$ and the sum. How can we continue?Now we have to transform the sum' in a integral. The density of state depends of $kdk$ so i think we have to evaluate: $$\int\limits_0^\infty {\frac{{e^{ - \frac{{\hbar ^2 k_a^2j }}{{2m}}} e^{ - \frac{{\hbar ^2 k_\gamma ^2k }}{{2m}}} e^{ - \frac{{\hbar ^2 k_\lambda ^2l }}{{2m}}} }}{{\frac{1}{2}\left( {k_a^2 + k_b^2 - k_\gamma ^2 - k_\lambda ^2 } \right)}}k_a k_b k_\gamma k_\lambda } dk_a dk_b dk_\gamma dk_\lambda $$ We have to evaluate two integrals, i think. Do you have any suggestion?


I have tried to solve this integral: $$F\left( {a,b,c,d} \right) = \int {\frac{{e^{ - \frac{{\hbar ^2 }}{{2m}}\left( {ak_a^2 + bk_b^2 + ck_\gamma ^2 + dk_\lambda ^2 } \right)} }}{{\left( {k_a^2 + k_b^2 - k_\gamma ^2 - k_\lambda ^2 } \right)}}k_a k_b k_\gamma k_\lambda dk_a dk_b dk_\gamma dk_\lambda } $$ from which we can calculate our integral putting $b=0$. We have: $$\frac{{\partial F}}{{\partial a}} + \frac{{\partial F}}{{\partial b}} - \frac{{\partial F}}{{\partial c}} - \frac{{\partial F}}{{\partial d}} \propto \frac{1}{{abcd}} $$ Now we put: $$i=a+b+c+d$$ $$l=a+b-c-d$$ $$m=a-b$$ $$n=c-d$$ So: $$\frac{\partial }{{\partial a}} = \frac{\partial }{{\partial i}} + \frac{\partial }{{\partial l}} + \frac{\partial }{{\partial m}} $$ $$\frac{\partial }{{\partial b}} = \frac{\partial }{{\partial i}} + \frac{\partial }{{\partial l}} - \frac{\partial }{{\partial m}} $$ $$\frac{\partial }{{\partial c}} = \frac{\partial }{{\partial i}} - \frac{\partial }{{\partial l}} + \frac{\partial }{{\partial n}} $$ $$\frac{\partial }{{\partial d}} = \frac{\partial }{{\partial i}} - \frac{\partial }{{\partial l}} - \frac{\partial }{{\partial n}} $$ $$\frac{\partial }{{\partial a}} + \frac{\partial }{{\partial b}} - \frac{\partial }{{\partial c}} - \frac{\partial }{{\partial d}} = 4\frac{\partial }{{\partial l}} $$ and: $$a=(i+l+2m)/4$$ $$b=(i+l-2m)/4$$ $$c=(i-l+2n)/4$$ $$d=(i-l-2n)/4$$ and the equation becomes: $$\frac{{\partial F'}}{{\partial l}} \propto \frac{1}{{\left( {\frac{{i + l}}{2} + m} \right)\left( {\frac{{i + l}}{2} - m} \right)\left( {\frac{{i - l}}{2} + n} \right)\left( {\frac{{i - l}}{2} - n} \right)}} $$ The integration can be evaluated with derive, and the substitutions gives an expression divergent if we put $b=0$. What is wrong?

$\endgroup$
2

1 Answer 1

0
$\begingroup$

Think of $k_\alpha$ and $k_\beta$ as $x$ and $y$, so $dk_\alpha\;dk_\beta = dx\;dy.$ Now convert to polar coordinates = $rdr\;d\theta$. The integral over theta is trivial. Repeat with the other two wave vectors and you end up with two integrals.

$\endgroup$
10
  • $\begingroup$ Hey, do you want me to actually have to type the trivial calculation in? Herp derp. $\endgroup$ Commented Nov 20, 2011 at 5:52
  • $\begingroup$ Sorry i respond you soo late.Can you give me the calculations??? $\endgroup$
    – Pablo
    Commented Dec 27, 2011 at 21:58
  • $\begingroup$ There's no calculation to type in. Simply replace $dk_\alpha\;dk_\beta = r\;dr\;d\theta$ where $k_\alpha$ and $k_\beta$ run from between + and - infinity, while r runs from 0 to infinity and theta runs from 0 to 2 pi. This is the usual Cartesian to polar conversion. $\endgroup$ Commented Jan 6, 2012 at 15:48
  • $\begingroup$ The result is very difficul to write, i don't think it's soo easy to get it. I tried to use polar coordinates but i don't know how to integrate! $\endgroup$
    – Pablo
    Commented Jan 6, 2012 at 23:27
  • $\begingroup$ Try using the substitution $\int_{-\infty}^{+\infty}\int_{-\infty}^{+\infty} H(k_\alpha,k_\beta)\;dk_\alpha\;dk_\beta$ $= \int_{r=0}^\infty\int_{\theta=0}^{2\pi}H(r\cos(\theta),r\sin(\theta))r\;d\theta \; dr$ $= \int_{r=0}^\infty\int_{\theta=0}^{2\pi}H(r) r\;d\theta \; dr$ $= 2\pi \int_{r=0}^\infty H(r)\;r \;dr$. This is a method of simplifying an integral of H that depends on the requirement that H simplify. It reduces two integrals down to 1. Also note that if the original integral of H was from 0 to infinity instead of -infinity to +infinity, then you have to make some simple changes. $\endgroup$ Commented Jan 8, 2012 at 5:46

Your Answer

By clicking “Post Your Answer”, you agree to our terms of service and acknowledge you have read our privacy policy.

Not the answer you're looking for? Browse other questions tagged or ask your own question.